Proving \{ \gamma^5 , \gamma^\mu \} = 0: A Guide to Gamma 5 Matrix Verification

So if you have, for example, L = L(q, \dot q) and q is a function of q and p, then \dot q = \dot q(q, p) and the Hamiltonian is H(q, p) = \dot q \frac{\partial L}{\partial \dot q} - L(q, \dot q).
  • #1
latentcorpse
1,444
0
How do I verify [itex] \{ \gamma^5 , \gamma^\mu \} = 0[/itex]

I have

[itex] \{ \gamma^5 , \gamma^\mu \} = \gamma^5 \gamma^\mu + \gamma^\mu \gamma^5[/itex]
[itex]= -i ( \gamma^0 \gamma^1 \gamma^2 \gamma^3 \gamma^5 + \gamma^5 \gamma^0 \gamma^1 \gamma^2 \gamma^3 )[/itex]
[itex] = -i ( \gamma^0 \gamma^1 \gamma^2 \gamma^3 \gamma^5 - \gamma^0 \gamma^5 \gamma^1 \gamma^2 \gamma^3 )[/itex]
[itex] = -i ( \gamma^0 \gamma^1 \gamma^2 \gamma^3 \gamma^5 + \gamma^0 \gamma^1 \gamma^5 \gamma^2 \gamma^3 )[/itex]
[itex] = -i ( \gamma^0 \gamma^1 \gamma^2 \gamma^3 \gamma^5 - \gamma^0 \gamma^1 \gamma^2 \gamma^5 \gamma^3 )[/itex]
[itex] = -i ( \gamma^0 \gamma^1 \gamma^2 \gamma^3 \gamma^5 + \gamma^0 \gamma^1 \gamma^2 \gamma^3 \gamma^5 )[/itex]

But this is not quite right, because at some point I will have shifted the [itex]\gamma^\mu[/itex] past itself and so I will get an additional term [itex]+2 \eta^{ \mu \mu}[/itex] since [itex] \{ \gamma^\mu , \gamma^\nu \} = 2 \eta^{\mu \nu}[/itex]

So I should get three terms:
[itex] = -i ( \gamma^0 \gamma^1 \gamma^2 \gamma^3 \gamma^5 + \gamma^0 \gamma^1 \gamma^2 \gamma^3 \gamma^5 -2 \eta^{\mu \mu} \gamma^0 \gamma^1 \gamma^2 \gamma^3 \gamma^5 )[/itex]

and then

[itex] = -i ( (2-2 \eta^{\mu \mu}) \gamma^0 \gamma^1 \gamma^2 \gamma^3 \gamma^5) \neq 0[/itex] since [itex]\eta^{\mu \mu} = 4[/itex], no?
 
Physics news on Phys.org
  • #2
So you have to prove that
[tex]
\gamma^0 \gamma^1 \gamma^2 \gamma^3 \gamma^{\mu} + \gamma^{\mu} \gamma^0 \gamma^1 \gamma^2 \gamma^3 = 0
[/tex]

The main idea is to get matrices with identical indexes close to each other.
Note that
1. every time you switch two [itex]\gamma[/itex]'s with [itex]\mu \neq \nu[/itex] you get a minus sign
2. if you have to move [itex]\gamma^\mu[/itex] in the first summand k times to the left, you will have to move it in the second summand (3-k) times to the right, thus giving ou additional (-1)
 
  • #3
quZz said:
So you have to prove that
[tex]
\gamma^0 \gamma^1 \gamma^2 \gamma^3 \gamma^{\mu} + \gamma^{\mu} \gamma^0 \gamma^1 \gamma^2 \gamma^3 = 0
[/tex]

The main idea is to get matrices with identical indexes close to each other.
Note that
1. every time you switch two [itex]\gamma[/itex]'s with [itex]\mu \neq \nu[/itex] you get a minus sign
2. if you have to move [itex]\gamma^\mu[/itex] in the first summand k times to the left, you will have to move it in the second summand (3-k) times to the right, thus giving ou additional (-1)

Thanks. Can I also ask, why equation 3.25 is correct? Isn't that the interaction Lagrangian instead of the interaction Hamiltonian? I think you'll need to look at eqn 3.7 as well.

Cheers.
 
  • #4
em... which book? =)
 
  • #6
It comes right from the connection between hamiltonian and lagrangian.

If you have for lagrangian L = L1 + L2 and L2 does not depend on derivatives, then for hamiltonian you have H = H1 + H2, where H1 corresponds to L1 and H2 = -L2.

Even more, if you have infinitesimal addition (that can depend on derivatives) to lagrangian: L = L0 + L', then for hamiltonian H = H0 + H', where H0 corresponds to L0 and H' = -L'.
 
  • #7
quZz said:
It comes right from the connection between hamiltonian and lagrangian.

If you have for lagrangian L = L1 + L2 and L2 does not depend on derivatives, then for hamiltonian you have H = H1 + H2, where H1 corresponds to L1 and H2 = -L2.

Even more, if you have infinitesimal addition (that can depend on derivatives) to lagrangian: L = L0 + L', then for hamiltonian H = H0 + H', where H0 corresponds to L0 and H' = -L'.

when you say H0 correspsonds to L0, do you mean H0=L0? Or it is just some function of L0?
 
  • #8
H0 can't be equal to L0, because they don't depend on the same (field) variables. For the system without constraints, the Hamiltonian is the Legendre transformation of the Lagrangian wrt the generalized velocities.
 

What is a Gamma 5 Matrix?

A Gamma 5 Matrix is a mathematical construct used in the field of quantum mechanics to represent the spin state of a particle. It is a 4x4 matrix that contains complex numbers and is used to describe the behavior of fermions, such as electrons, in quantum systems.

What is the significance of the number 5 in the Gamma 5 Matrix?

The number 5 in the Gamma 5 Matrix refers to the fifth component of the Dirac gamma matrices, which are used to represent the spin and momentum of particles in quantum mechanics. The Gamma 5 Matrix is unique in that it is the only one that is purely imaginary and has a zero trace.

How is the Gamma 5 Matrix related to the Dirac equation?

The Gamma 5 Matrix is a fundamental part of the Dirac equation, which is a relativistic wave equation that describes the behavior of fermions in quantum systems. The Gamma 5 Matrix is used to represent the spinor field in the equation and is essential for understanding the properties and behavior of particles at the quantum level.

What are the applications of the Gamma 5 Matrix?

The Gamma 5 Matrix has many applications in the field of quantum mechanics, including in the study of particle physics, quantum field theory, and condensed matter physics. It is also used in the development of quantum computing and in the understanding of the behavior of particles in high-energy collisions.

How is the Gamma 5 Matrix represented in mathematical notation?

In mathematical notation, the Gamma 5 Matrix is typically represented as γ5 or γ5. It can also be written as γ0γ1γ2γ3, as it is a product of the other four Dirac gamma matrices.

Similar threads

  • Advanced Physics Homework Help
Replies
8
Views
712
  • Advanced Physics Homework Help
Replies
20
Views
2K
  • Advanced Physics Homework Help
Replies
0
Views
265
  • Advanced Physics Homework Help
Replies
18
Views
2K
  • Advanced Physics Homework Help
Replies
2
Views
441
  • Advanced Physics Homework Help
Replies
7
Views
901
  • Advanced Physics Homework Help
Replies
25
Views
3K
  • Advanced Physics Homework Help
Replies
1
Views
1K
  • Advanced Physics Homework Help
Replies
20
Views
2K
  • Advanced Physics Homework Help
Replies
0
Views
998
Back
Top